A VWAP system simulated performance

Hi Glen,
I confess that I've always struggled with the concept of ATR, so your question is best answered by Iraj, Paul or someone more proficient in its use than me. That said, it appears from your excel doc. that you're using daily ATR for an intra day trade. So, in the case of trade 1, day 1, AYI has an ATR of $2.00. If the entry price is $38.01, then your stop loss would have been at $36.01 (assuming 1 x ATR). On this basis, your position size would have been 150 shares. (150 x $2.00 = $300.00, i.e. 1% of total equity.) Clearly the position size is too small, so I would have thought that the ATR ought to be measured by the intra day timeframe you're using. If the trades are open and closed in the last two hour session, then I would have thought that it should be based on a maximum of 15 minute ATR. As I say, this isn't my field and I'm floating these ideas in the hopes that I'll get a better understanding from those that know what they're doing!
;)
Tim.

Thansk for your comments Tim,

as you say working it with 1% of account balance would produce a very small position size, i'm not sure this would be suitable for a strat like this

i still need to do some work to get my head around using ATR for risk as well

Glen
 
Correct position sizing


$30K account balance

i want to risk say 20% of account bal per trade therefore

shorts

CY 35.99 ATR 15min = 30cents (risk 2 x ATR), therefore position size 278 ???
GES 45.66 ATR 15min = 37cents (risk 2 x ATR), therefore position size 178 ???
PLCM 24.07 ATR 15min = 6cents (risk 2 x ATR), therefore position size 2077 ???

longs

IAR 16.66 ATR 15min = 7.5cents (risk 2 x ATR), therefore position size 2401 ???
LCC 18.37 ATR 15min = 10cents (risk 2 x ATR), therefore position size 1633 ????
ABK 24.04 ATR 15min = 112cents (risk 2 x ATR), therefore position size 111 ????

comments welcome not sure if this is correct postion sizing
 
$30K account balance

i want to risk say 20% of account bal per trade therefore

shorts

CY 35.99 ATR 15min = 30cents (risk 2 x ATR), therefore position size 278 ???
GES 45.66 ATR 15min = 37cents (risk 2 x ATR), therefore position size 178 ???
PLCM 24.07 ATR 15min = 6cents (risk 2 x ATR), therefore position size 2077 ???

longs

IAR 16.66 ATR 15min = 7.5cents (risk 2 x ATR), therefore position size 2401 ???
LCC 18.37 ATR 15min = 10cents (risk 2 x ATR), therefore position size 1633 ????
ABK 24.04 ATR 15min = 112cents (risk 2 x ATR), therefore position size 111 ????

comments welcome not sure if this is correct postion sizing

Glen,
I think the calculation should be along the lines off ...

Capital = $30,000 Risk = 1% therefore risk per trade is $300

CY 35.99 ATR 15min = 30cents (risk 2 x ATR = $0.60), therefore position size 300/0.6 = 500 shares

Jonathan
 
Glen,
I think the calculation should be along the lines off ...

Capital = $30,000 Risk = 1% therefore risk per trade is $300

CY 35.99 ATR 15min = 30cents (risk 2 x ATR = $0.60), therefore position size 300/0.6 = 500 shares

Jonathan

Hi Jonathan,

thanks for the correction (can safely say i made a balls of the above example)

will re-do

was going to used 20% risk per trade (with margin) (as market neutral) not 1%

cheers
Glen
 
$30K account balance

i want to risk say 20% of account bal per trade therefore

shorts

CY 35.99 ATR 15min = 30cents (risk 2 x ATR), therefore position size 278 ???
GES 45.66 ATR 15min = 37cents (risk 2 x ATR), therefore position size 178 ???
PLCM 24.07 ATR 15min = 6cents (risk 2 x ATR), therefore position size 2077 ???

longs

IAR 16.66 ATR 15min = 7.5cents (risk 2 x ATR), therefore position size 2401 ???
LCC 18.37 ATR 15min = 10cents (risk 2 x ATR), therefore position size 1633 ????
ABK 24.04 ATR 15min = 112cents (risk 2 x ATR), therefore position size 111 ????

comments welcome not sure if this is correct postion sizing

account balance $30K

i want to risk say 20% of account bal per trade therefore

shorts

CY 35.99 ATR 15min = 30cents (risk 2 x ATR),

Risk on trade = $30K x 20% = $6K / .60 = 10,000 shares


GES 45.66 ATR 15min = 37cents (risk 2 x ATR),

Risk on trade = $30K x 20% = $6K / .74 = 8,108 shares


PLCM 24.07 ATR 15min = 6cents (risk 2 x ATR),

Risk on trade = $30K x 20% = $6K / .12 = 50,000 shares


longs

IAR 16.66 ATR 15min = 7.5cents (risk 2 x ATR),

Risk on trade = $30K x 20% = $6K / .15 = 40,000 shares


LCC 18.37 ATR 15min = 10cents (risk 2 x ATR),

Risk on trade = $30K x 20% = $6K / .2 = 30,000 shares


ABK 24.04 ATR 15min = 112cents (risk 2 x ATR),

Risk on trade = $30K x 20% = $6K / 2.24 = 2,679 shares



lol! think I may be over doing it a bit with 20% risk

will re-do again!
 
Last edited:
account balance $30K

i want to risk say 20% of account bal per trade therefore

shorts

CY 35.99 ATR 15min = 30cents (risk 2 x ATR),

Risk on trade = $30K x 20% = $6K / .60 = 10,000 shares


GES 45.66 ATR 15min = 37cents (risk 2 x ATR),

Risk on trade = $30K x 20% = $6K / .74 = 8,108 shares


PLCM 24.07 ATR 15min = 6cents (risk 2 x ATR),

Risk on trade = $30K x 20% = $6K / .12 = 50,000 shares


longs

IAR 16.66 ATR 15min = 7.5cents (risk 2 x ATR),

Risk on trade = $30K x 20% = $6K / .15 = 40,000 shares


LCC 18.37 ATR 15min = 10cents (risk 2 x ATR),

Risk on trade = $30K x 20% = $6K / .2 = 30,000 shares


ABK 24.04 ATR 15min = 112cents (risk 2 x ATR),

Risk on trade = $30K x 20% = $6K / 2.24 = 2,679 shares



lol! think I may be over doing it a bit with 20% risk

will re-do again!


account balance $30K

i want to risk say 5% of account bal per trade therefore

shorts

CY 35.99 ATR 15min = 30cents (risk 2 x ATR),

Risk on trade = $30K x 5% = $1,5K / .60 = 2,500 shares


GES 45.66 ATR 15min = 37cents (risk 2 x ATR),

Risk on trade = $30K x 5% = $1,5K / .74 = 2027 shares


PLCM 24.07 ATR 15min = 6cents (risk 2 x ATR),

Risk on trade = $30K x 5% = $1,5K / .12 = 12,500 shares


longs

IAR 16.66 ATR 15min = 7.5cents (risk 2 x ATR),

Risk on trade = $30K x 5% = $1,5K/ .15 = 10,000 shares


LCC 18.37 ATR 15min = 10cents (risk 2 x ATR),

Risk on trade = $30K x 5% = $1,5K// .2 = 7500shares


ABK 24.04 ATR 15min = 112cents (risk 2 x ATR),

Risk on trade = $30K x 5% = $1,5K / 2.24 = 770shares


still way to big!!!!!! (will re-do at 1%, see what it look like)
 
account balance $30K

i want to risk say 5% of account bal per trade therefore

shorts

CY 35.99 ATR 15min = 30cents (risk 2 x ATR),

Risk on trade = $30K x 5% = $1,5K / .60 = 2,500 shares


GES 45.66 ATR 15min = 37cents (risk 2 x ATR),

Risk on trade = $30K x 5% = $1,5K / .74 = 2027 shares


PLCM 24.07 ATR 15min = 6cents (risk 2 x ATR),

Risk on trade = $30K x 5% = $1,5K / .12 = 12,500 shares


longs

IAR 16.66 ATR 15min = 7.5cents (risk 2 x ATR),

Risk on trade = $30K x 5% = $1,5K/ .15 = 10,000 shares


LCC 18.37 ATR 15min = 10cents (risk 2 x ATR),

Risk on trade = $30K x 5% = $1,5K// .2 = 7500shares


ABK 24.04 ATR 15min = 112cents (risk 2 x ATR),

Risk on trade = $30K x 5% = $1,5K / 2.24 = 770shares


still way to big!!!!!! (will re-do at 1%, see what it look like)



account balance $30K

i want to risk say 1% of account bal per trade therefore

shorts

CY 35.99 ATR 15min = 30cents (risk 2 x ATR),

Risk on trade = $30K x 1% = $300 / .60 = 500 shares


GES 45.66 ATR 15min = 37cents (risk 2 x ATR),

Risk on trade = $30K x 1% = $300 / .74 = 405 shares


PLCM 24.07 ATR 15min = 6cents (risk 2 x ATR),

Risk on trade = $30K x 1% = $300 / .12 = 2,500 shares


longs

IAR 16.66 ATR 15min = 7.5cents (risk 2 x ATR),

Risk on trade = $30K x 1% = $300/ .15 = 2000 shares


LCC 18.37 ATR 15min = 10cents (risk 2 x ATR),

Risk on trade = $30K x 1% = $300// .2 = 1500shares


ABK 24.04 ATR 15min = 112cents (risk 2 x ATR),

Risk on trade = $30K x 1% = $300 / 2.24 = 133 shares


looking better!
 
Glen,

The risk should be applied against your capital and the margin you have allows for the resulting postion size. We need to remember that 500 shares @ $36 is $18k and you are trading a basket of stocks that will add up to something scary :LOL:

I'm only para phrasing stuff I've read on here and I'd still like to know myself how to position size a market neutral strategy.

Thanks, Jonathan
 
account balance $30K

i want to risk say 1% of account bal per trade therefore

shorts

CY 35.99 ATR 15min = 30cents (risk 2 x ATR),

Risk on trade = $30K x 1% = $300 / .60 = 500 shares


GES 45.66 ATR 15min = 37cents (risk 2 x ATR),

Risk on trade = $30K x 1% = $300 / .74 = 405 shares


PLCM 24.07 ATR 15min = 6cents (risk 2 x ATR),

Risk on trade = $30K x 1% = $300 / .12 = 2,500 shares


longs

IAR 16.66 ATR 15min = 7.5cents (risk 2 x ATR),

Risk on trade = $30K x 1% = $300/ .15 = 2000 shares


LCC 18.37 ATR 15min = 10cents (risk 2 x ATR),

Risk on trade = $30K x 1% = $300// .2 = 1500shares


ABK 24.04 ATR 15min = 112cents (risk 2 x ATR),

Risk on trade = $30K x 1% = $300 / 2.24 = 133 shares


looking better!


total funds used in underlining = $160,734 (would need margin relief for being neutal, as IB account has only 25% on US stocks)


500 shares x 35.99 = $17,995
405 shares x 45.66 = $18,492
2,500 shares x 24.07 = $60,175

2000 shares x 16.66 = $33,320
1500shares x 18.37 = $27,555
133 shares x 24.04 = $3197

getting there...:|
 
Last edited:
Glen,

The risk should be applied against your capital and the margin you have allows for the resulting postion size. We need to remember that 500 shares @ $36 is $18k and you are trading a basket of stocks that will add up to something scary :LOL:

I'm only para phrasing stuff I've read on here and I'd still like to know myself how to position size a market neutral strategy.

Thanks, Jonathan

J,

was just writing the above post at the same time as you were writing this,

agree with you totally here.:cool:

Glen
 
won't post results sheet tonight until i get input from Grey1 or Trader333 on the above position sizing model based on ATR.

I've calculated tonight results for both the old position size model (the crap one i made up) which didn't take ATR into account and the new model based on ATR 15min.

if the new risk model based on ATR is correct then i will re-set the test to Day 1 from tonight and continue with the test.

I don't believe posting the results P/L in cents make sense :|
and I don't believe posting the results in $$ using the old position sizing model (the crap one i made up) is a true test either

every days a school day on TT:|

ta
Glen
 
Last edited:
won't post results sheet tonight until i get input from Grey1 or Trader333 on the above position sizing model based on ATR.

I've calculated tonight results for both the old position size model which didn't take ATR into account and the new model based on ATR 15min.

if the new risk model based on ATR is correct then i will re-set the test to Day 1 from tonight and continue with the test.

I don't believe posting the results P/L in cents make sense :|
and I don't believe posting the results in $$ using the old position sizing model (the crap one i made up) is a true test either

every days a school day on TT:|

ta
Glen

Best thing is to get yourself the TS and the code from Paul / ATR changes with volatilty and you need to have a correct pos size ,,, have you got TS yet ? let me know when you are fixed with TS so i can send you a code to help you out in choosing the correct stocks ..



Grey1
 
Best thing is to get yourself the TS and the code from Paul / ATR changes with volatilty and you need to have a correct pos size ,,, have you got TS yet ? let me know when you are fixed with TS so i can send you a code to help you out in choosing the correct stocks ..



Grey1

Have got TS2i,

Haven't got the link software (hyerserver) to TWS yet

will have soon

thanks, look forward to the code (save a lot of assing about, althought i've learnt alot in the last few days..)
 
Running totals (Day 9)

worked back through previous results and applied position sizing based on ATR.


View attachment 31627




P/L for today day 9


-$1,926 (worst night yet)

account balance = $26,603 (starting account $30,000)
 
Last edited:
As far as I understand, this strategy is the mean reversion to VWAP. That means profit target = the distance between entry price and VWAP. (For Long stocks, estimated return = VWAP - entry price, and for Short, return = entry price - VWAP) . We can not expect any more than that as the time is trade is less than 2 hours and the chosen stocks would be strongly trending stocks. ( That means we can not expect the strong stocks to fall from high of the day to low of the day and the reverse for weak stocks). So the expected return is the distance to VWAP for both strong and weak stocks.

For most strategeis, the the expected return should be greater than or equal to the risk taken unless the winning rate is very high (say 75% or above). It looks like the objective of this exercise is to prove that this is a high probability strategy. And stops should be set at contingency level only and they are not not meant to be triggered unless there is a big directional move. ( large stops, 2 hr trade with Daily ATR ), due to the fact that it is a pair trading with correct position sizing. Therefore, I think the Daily ATR is to be used primarily for position sizing.
 
Thanks for your input

lets have an exercise here,,, around 6 PM UK time choose 3 stocks that are in positive territory and up by say 5% and 3 stocks that are below by 5% .. now short the strong ones (those which are above by 5% ) and long the weak ones ( excuse me about using the word WEAK and STRONG as there is more to this phrase in TA ) ,, Do this for 5 days and some one post the results here, NO excuse NO emotion just post the results and surprise me if i was wrong ,,, remember statistical approach to day trading might not work in very short period but I feel 5 days is good enough )

Grey1
.

Grey1's post above is why i started this, and also to learn about VWAP strat, position sizing etc

As far as I understand, this strategy is the mean reversion to VWAP. That means profit target = the distance between entry price and VWAP. (For Long stocks, estimated return = VWAP - entry price, and for Short, return = entry price - VWAP) . .

i'm not looking at VWAP yet, just mean reverion as requested above

We can not expect any more than that as the time is trade is less than 2 hours and the chosen stocks would be strongly trending stocks. ( That means we can not expect the strong stocks to fall from high of the day to low of the day and the reverse for weak stocks). So the expected return is the distance to VWAP for both strong and weak stocks.

this could be correct, but i don't have the experince in VWAP or US stocks to comment (will hopefully soon tho)

For most strategeis, the the expected return should be greater than or equal to the risk taken unless the winning rate is very high (say 75% or above). It looks like the objective of this exercise is to prove that this is a high probability strategy. And stops should be set at contingency level only and they are not not meant to be triggered unless there is a big directional move. ( large stops, 2 hr trade with Daily ATR ), due to the fact that it is a pair trading with correct position sizing. Therefore, I think the Daily ATR is to be used primarily for position sizing.

Grey1, says somewhere on this thread to use 2 x ATR 10mins to place stops.. i then used this for position sizing
(could be right could be wrong)

thansk

Glen
 
Last edited:
Top